The price of a pair of shoes after 8% sales tax was added was $77.76. Find the original price before sales tax

Answers

Answer 1

Answer: $71.53

Step-by-step explanation:

77.76 = 100%


Related Questions

Find x in the equation. 4 times x plus one half equals one half times x plus 4

Answers

After solving the equation, the value obtained for x will be equal to 1.

What is an equation?

Mathematical expressions with two algebraic symbols on either side of the equal (=) sign are called equations.

This relationship is illustrated by the left and right expressions being equal. The left-hand side equals the right-hand side is a basic, straightforward equation.

As per the information given in the question,

4 times x plus one-half equals one-half times x plus 4.

Let's write it in mathematical form,

4x + 1/2 = 1/2x + 4

Now, solve the equation for x,

4x - 1/2x = 4 - 1/2

(8x - x)/2 = (8 - 1)/2

7x/2 = 7/2

x = (7 × 2)/(2 × 7)

x = 14/14

x = 1

To know more about equation:

https://brainly.com/question/29657983

#SPJ1

The Tri-State Pick 3 lottery game offers a choice of several bets. You choose
a three-digit number. The lottery commission announces the winning three-digit number,
chosen at random, at the end of each day. The "box" pays $83.33 if the number you choose
has the same digits as the winning number, in any order. Find the expected payoff for a
$1 bet on the box. (Assume that you chose a number having three different digits.)

Answers

Answer: The Tri-State Pick 3 lottery game offers a choice of several bets, including a "box" bet where you choose a three-digit number and win if the number you choose has the same digits as the winning number, in any order. The expected payoff for a $1 bet on the box is $83.33.

To find this expected value, we need to consider all the possible ways that the winning number could be chosen and calculate the probability of each outcome. Since the winning number is chosen at random, each of the 10 possible digits (0 through 9) is equally likely to be chosen for each of the three positions in the winning number.

Since the box bet pays off if the digits of your chosen number match the digits of the winning number, regardless of their order, the number of winning combinations is equal to the number of ways to arrange the three digits of the winning number. This is equal to 3! (3 factorial), which is equal to 6.

Since there are a total of 10 possible digits for each position and the winning number has three positions, there are a total of 10 x 10 x 10 = 1000 possible winning numbers. Of these, 6 will pay off for a box bet, so the probability of winning a box bet is 6/1000 = 0.006.

Since the expected value of a bet is equal to the probability of winning multiplied by the payoff, the expected value of a $1 bet on the box is 0.006 x $83.33 = $0.05. Therefore, the expected payoff for a $1 bet on the box is $0.05.

Express 80 as the product of its prime factors.
Write the prime factors in ascending order.

Answers

Answer: [tex]2^4 \times 5[/tex]

Step-by-step explanation:

[tex]80=2 \times 40\\\\80=2 \times 2 \times 20\\\\80=2 \times 2 \times 2 \times 10\\\\80=2 \times 2 \times 2 \times 2 \times 5\\\\80=2^4 \times 5[/tex]

Ms. Hall is buying Ms. Ward some gifts for Christmas. She is buying one big gift that
will cost $90 as well as some small gifts that cost $7 each.
1+90 +7
What is its :
Constant:
Coefficient
Equation:

Answers

When Ms. Hall is buying Ms. Ward some gifts for Christmas as described in the problem,

the Constant: $90 and $7

the Coefficient: $7

the Equation:  $90 + $7x

How to find the constant

Information form the problem shows that

one big gift that will cost $90

some small gifts that cost $7 each

the constant in this case is the price of the gifts

$90 and $7

the coefficient is $7

the equation is

$90 + $7x

where x is the number of the small gifts

Learn more about equations at

https://brainly.com/question/22688504
#SPJ1


Fátima, Gabriel y José tienen un total de $1080 en sus carteras. Gabriel tiene 3 veces lo que José tiene. Fátima tiene $70 menos
que José. ¿Cuanto dinero tienen en sus carteras?

Answers

Fátima tiene un total de $ 160, Gabriel tiene un total de $ 690 y José tiene un total de $ 230.

¿Cómo determinar el dinero requerido en sus carteras?

En este problema tenemos a tres personas, Fátima, Gabriel y José, que reúnen un total de $ 1.080. A continuación, traducimos las variables y ecuaciones asociadas a la situación:

x - Cantidad de dinero de Fátima.

y - Cantidad de dinero de Gabriel.

z - Cantidad de dinero de José.

Ecuaciones: Fátima, Gabriel y José tienen un total de $ 1080.

x + y + z = 1080

Gabriel tiene 3 veces lo que José tiene.

y = 3 · z

Fátima tiene $ 70 menos que José.

x = z - 70

A continuación, resolvemos el sistema de ecuaciones lineales:

x + y + z = 1080

(z - 70) + 3 · z + z = 1080

5 · z - 70 = 1080

5 · z = 1150

z = 1150 / 5

z = 230

x = z - 70

x = 230 - 70

x = 160

y = 3 · z

y = 3 · 230

y = 690

Para aprender más de sistemas de ecuaciones lineales: https://brainly.com/question/29142634

#SPJ1

Julie buys a pair of earrings for $7. Now she would like to buy the same earrings for 2 of her friends. How much will she spend for all 3 pairs of earrings

Answers

$21 because 7x3 is 21
7•3= 21 therefore the answer is $21

Blaine is going bowling and will need to
pay $4 for shoes and then $3 for every
game. Write an equation for the
situation, if Blaine spends 16 dollars.

Answers

Answer: $16 = 4s + 3g

Step-by-step explanation:

s = shoes

g = games

Is this true or false?

Answers

TRUE: The given contrapositive statement are correct,

If m ≤ 3, then, m < 5.

Explain the contrapositive statement?An inference rule called proof by contrapositive, also known as proof by contraposition, is used in proofs to infer a conditional assertion from its contrapositive. To put it another way, the assertion "if B, then not A" is proven rather than the conclusion "if A, then B."When you reverse the hypothesis as well as the conclusion in a statement and reject both of them, you have a contrapositive statement.

For the stated conditional statement -

If m ≤ 3, then, m < 5.

Then, contrapositive statement for the given statement will be

(m < 5)' ⇒ (m ≤ 3)'

That is;

(m > 5) ⇒ (m > 3)

That is, if m> 5 then, m > 3, which is true.

Thus, the contrapositive statement for the given conditional statement is true.

To know more about the contrapositive statement, here

https://brainly.com/question/1713053

#SPJ1

1. Which of the following is a perfect cube. (A) 100 (B) 68600 (C) 343000 (D) 400​

Answers

Answer: 68600

hope it helps!

68600 is the answer I think!

Which stataments are true regarding undifinable terms in geometry?

Answers

Answer:

A point has no length or widthA line consists of an infinite number of pointsA plane consists of an infinite set of lines

Step-by-step explanation:

The statements that are true regarding undefined terms are;

A point has no length or widthA line consists of an infinite number of pointsA plane consists of an infinite set of lines

The reason these statements above are true are as follows:

Examples of undefined terms includes a point, a plane, a line and a set

A point is for representing a position in space. It has no dimensions

Therefore, the statement, A point has no length or width is true

A plane is defined as an interconnected set of points that lay on the same flat surface, and extends infinitely along the surface in all directions

A line is defined as an infinite set of points that are arranged in straight path and extends without end in opposite directions

Therefore, the statement, a line consists of an infinite number of points is true

Therefore, a plane can be considered as being made up of an infinite set of lines that are located on a flat surface

Therefore, the statement, a plane consists of an infinite set of lines is true

Help help help help help

Answers

Net profit is a company's profit after all expenses have been deducted. What remains after deducting fixed costs from gross profit is the net profit.

Why do we calculate net profit?

Subtracting the cost function from the revenue function yields the profit function. Assume that the profit is denoted by P(x), the revenue is denoted by R(x), the cost is denoted by C(x), and x is the quantity of goods sold. Consequently, the profit function is denoted by P(x) = R(x0 - C) (x).

Total Revenue - Total Expenses = Net Profit

A company that sells online has a $350,000 revenue and a $50,000 cost of products sold.The net income formula is defined as the difference between the total revenue earned and the total costs.Total revenue minus all costs equals net income.

The money a corporation makes after deducting interest, operational costs, and taxes over a specific time period is known as net profit.Profit After Tax is another name for Net Profit (PAT).

To learn more about net profit refer to :

https://brainly.com/question/13788133

#SPJ1

Which of the following choices is not an example of a unit rate?
earnings per hour dollars per pound calories per serving $25.50 for 5 gallons

Answers

Examples of unit rates are Dollars per pound calories and earnings per hour. The rest is not the unit rate.

What is the unit rate?

An item's unit rate is its price for one of them. This is expressed as a ratio with a one as the denominator. For instance, if you covered 70 yards in 10 seconds, you covered 7 yards on average every second. 70 yards in 10 seconds and 7 yards in 1 second are both ratios, although only the latter is a unit rate.

Earnings per hour are a unit rate since it is calculated for an hour.

Dollars per pound calories is a unit rate.

Serving $25.50 for 5 gallons is not a unit rate, the cost is for 5 gallons. If the cost is for 1 gallon then it will unit rate.

To learn more about rate, click on the below link:

https://brainly.com/question/29781084

#SPJ1

help i need perimeter of these

Answers

Answer:  A= 52cm  B= 94mm  C= 9m  D= 17cm

Step-by-step explanation:

A) You first need to find the missing sides by doing

11-9= 2

and 15-9= 6

you then add it all together

6+2+9+9+15+11=52cm

B) You first need to find the missing sides by doing

25-18=7

and 22-6= 16

you then add it all together

25+22+18+16+7+6=94mm

C) You first need to find the missing sides by doing

0.5+1=1.5 then 2.5-1.5=1

and 1.2+0.8=2 then 2-0.5=1.5

you then add the numbers all together

1+1.5+0.8+1+0.5+0.5+1.2+2.5=9m

D) You first need to find the missing sides by doing

2.5+1= 3.5 then 3.5-2=1.5

and 1.5+2.5=4 then 5-4=1

you then add all the numbers together

2.5+2.5+1.5+1+5+2+1+1.5=17cm

Pleaseeee help I’m so confused

Answers

Answer:

D

Step-by-step explanation:

if you move the preimage on the image you can see that the two figures are equals

Please help me with this question!

Answers

Answer:

Step-by-step explanation:

the answer is 11/12

a doctor claims that people are at most 6 pounds overweight. to test the claim, the doctor randomly selects 81 people and calculates the difference between their ideal weight and actual weight. the sample mean and sample standard deviation of that difference were 7 pounds and 2 pounds respectively. can we conclude at the 1% level of significance that the claim is true? group of answer choices yes, conclude the claim is true by rejecting h0. no, conclude the claim is not true by rejecting h0. no, conclude the claim is not true by accepting h0. yes, conclude the claim is true by accepting h0.

Answers

we conclude at the 1% level of significance that the claim is true by accepting H₀

What is Hypothesis?

A hypothesis outlines your expectations for the results of your investigation. It is a speculative, untested response to your research question.

Given,

n = 81

x bar = 7

s= 2

Here, a doctor claims that people are atmost 6 pound over weight

H₀ : μ ≤ 6 versus

H₀ : μ > 6 versus

Now,

The Test Stastistic is

t = x bar - μ / s√n

t = 7-6/2√81

t = 4.5

let ∝ = 0.10, then p value = 0.000

p value >∝

yes, conclude the claim is true by accepting H₀

To learn more about Hypothesis visit:

brainly.com/question/11560606

#SPJ4

(x-14)÷7=12 I just need help

Answers

Answer:

x = 98

Step-by-step explanation:

Solve for x:

(x - 14)/7 = 12

Multiply both sides of (x - 14)/7 = 12 by 7:

(7 (x - 14))/7 = 7×12

(7 (x - 14))/7 = 7/7×(x - 14) = x - 14:

x - 14 = 7×12

7×12 = 84:

x - 14 = 84

Add 14 to both sides:

x + (14 - 14) = 14 + 84

14 - 14 = 0:

x = 84 + 14

84 + 14 = 98:

Answer:  x = 98

2) Given that Y and P are two events and P(Y) = 0.57, P(P) = 0.5 and P(Y UP) = 0.84,
Draw a Venn Diagram and find:
a) P(Yn P)
b) P(Y|P)
c) P(P|Y)
d) P(Y/P')

Answers

a) The value of P(Y∩P) =0.23

b) The value of (Y|P) = 0.34

c)  The value of (P|Y) = 0.27

d) The value of P(Y/P') = 0.68

What is Venn diagram?

A Venn diagram can also be referred to as a set diagram or a logic diagram that illustrates various set operations including the intersection, union, and difference of sets. Subsets of a set are also represented using it. A subset of whole numbers, which is a subset of integers, is a collection of natural numbers, as an example.

Given that there are two events Y and P.

P(Y) = 0.57

P(P) = 0.5

P(Y∪P) = 0.84

The formula of intersection is  P(Y∩P)  = P(P) + P(Y) - P(Y∪P)

Putting P(Y) = 0.57, P(P) = 0.5, P(Y∪P) = 0.84

P(Y∩P)  = 0.5 + 0.57 - 0.84

P(Y∩P) = 0.23.

b)

P(Y|P) = P(Y) - P(Y∩P)

P(Y|P) = 0.57 - 0.23

P(Y|P) = 0.34

c)

P(P|Y) = P(P) - P(Y∩P)

P(P|Y) = 0.5 - 0.23

P(P|Y) = 0.27

d) P(Y/P') = P(Y∩P') / P(P')

P(Y/P') = P(Y|P)/[1 - P(P)]

P(Y/P') = 0.34/(1- 0.5)

P(Y/P') = 0.68

To learn more about conditional probability, click on below link:

https://brainly.com/question/17888647

#SPJ1

1. The equation, 687.5 = 550(1 + 0.05t), represents the amount of money earned on a simple interest savings account. What does the value 550 represent?

The value 550 represents the initial balance in the account, which means the account started with $550.
The value 550 represents the final balance in the account, which means the account balance ended with $550.
The value 550 represents the final balance in the account, which means the account balance started with $550.
The value 550 represents the amount of interest earned, which means $550 was added to the initial balance.

2. A savings account is opened with an initial deposit of $425.00. Determine the account balance after 9 years if the account earns 2.7% simple interest annually.

$3,928.28
$1,457.75
$528.28
$103.28

3. An arithmetic sequence begins with 56, 59, 62, 65, 68, …

Which option below represents the formula for the sequence?

f(n) = 56 + 3(n − 1)
f(n) = 53 + 3(n − 1)
f(n) = 56 + 3(n)
f(n) = 53 + 3(n + 1)

4. The equation, 816 = 600(1 + 9r), represents the amount of money earned on a simple interest savings account. Solve for r.

r = 0.04
r = 0.14
r = 0.26
r = 0.40

Answers

Answer:

Answers below

Step-by-step explanation:

1. The value 550 represents the initial balance in the account, which means the account started with $550.

2. 528.28

425.00 x  2.7% x 9 = 103.275

Then we add 103.275 to the initial deposit of $425.00

425.00 + 103.275 = 528.275

Therefore the answer is $528.28

3. The answer for this one is  f(n) = 56 + 3(n − 1)

So, as we can see the first value is 56 and the common difference is 3 and the rule for the nth term is [tex]a_{n} =a_{1} +d(n-1)[/tex]

Therefore, its 56 + 3(n − 1)

4. For this one we solve for r and using a calculator its r = 0.04

The value of r from the given equation is 0.04. Therefore, option A is the correct answer.

What is the simple interest?

Simple interest is a method to calculate the amount of interest charged on a sum at a given rate and for a given period of time.

1) The given equation is 687.5=550(1+0.05t)

Here, amount=$687.5, principal (initial balance)=$550 and rate of interest =0.05×100 =5%

So, the value 550 represents the initial balance in the account, which means the account started with $550.

2) Given that, principal=$425.00, time period=9 years and rate of interest=2.7%

We know that, S.I=(P×T×R)/100

= (425×9×2.7)/100

= 103.275

Amount=103.275+425

= 528.275

≈ $528.28

3) The given arithmetic sequence is 56, 59, 62, 65, 68,.…

Here, first term (a)=56 and common difference=59-56=3

We know that, general form of arithmetic sequence is aₙ=a+(n-1)d

Now, f(n)=56+(n-1)3

4) The given equation is 816=600(1+9r)

1+9r =816/600

1+9r =1.36

9r=0.36

r=0.04

The value of r from the given equation is 0.04. Therefore, option A is the correct answer.

To learn more about the simple interest visit:

https://brainly.com/question/25845758.

#SPJ2

Cora plans to buy some calligraphy pens priced at $5 each. Write an equation that shows how the total cost, y, depends on the number of calligraphy pens Cora buys, x. Do not include dollar signs in the equation.

Answers

Answer: y = 5x

Step-by-step explanation:

How to solve this problem? I need the answer quick as possible!

Consider the following three systems of linear equations. Answer the questions below.

For each, choose the transformation and then fill in the blank with the correct number.

The arrow → means the expression on the left becomes the expression on the right.

Answers

Answer:

24 Inches

Step-by-step explanation:

The length of a piece of wood that uses 8 nails as required in the task content is; 24 inches.

What is the length of a piece of wood that uses 8 nails?

It follows from the task content that the length of a piece of wood that uses 8 nails is to be determined from the task content.

I need help please ok okokokookokook

Answers

Answer:

should be angle 7, 1, and 3

hope i could helpppp

Answer: 1 , 7 , 3

Step-by-step explanation:

The angles that habe the same measurement of angle 5 are;

1 , 7 , 3

Because the lines are parallel, these angles correspond to angle 5.

What is the smallest prime number that is the sum of two other distinct prime numbers?

Answers

Answer:

5

Step-by-step explanation:

2 +3 = 5    

in a hypothetical case-control study of female participants regarding exposure to emf's and breast cancer, the following data were obtained: 11 cases exposed, 108 controls exposed, 5 cases not exposed, 436 controls not exposed. what is the odds ratio for the association between exposure to emf's and breast cancer?

Answers

The odds ratio for the association between exposure to emf's and breast cancer would be (11/5) / (108/436) = 2.2.

1. Calculate the odds of exposure for cases: 11/5 = 2.2

2. Calculate the odds of exposure for controls: 108/436 = 0.25

3. Divide the odds of exposure for cases by the odds of exposure for controls: 2.2/0.25 = 8.8

4. Finally, the odds ratio for the association between exposure to EMF's and breast cancer is 2.2.

The odds ratio for the association between exposure to EMF's and breast cancer can be calculated by dividing the odds of exposure for cases by the odds of exposure for controls. To calculate this, first take the number of cases exposed (11) and divide it by the number of cases not exposed (5). This will give the odds of exposure for cases (2.2). Second, take the number of controls exposed (108) and divide it by the number of controls not exposed (436). This will give the odds of exposure for controls (0.25). Finally, divide the odds of exposure for cases (2.2) by the odds of exposure for controls (0.25). This will give the odds ratio for the association between exposure to EMF's and breast cancer (2.2).

Odds ratio = (11/5) / (108/436) = 2.2

Learn more about ratio here

https://brainly.com/question/13419413

#SPJ4

PLEASE ANSWER FAST

Find the missing values in the table below so that it represents a linear function.




Answers

The value of y₂, y₃ and y₄ in the linear function are -21, -26 and -31 respectively.

How to  represent linear function?

Linear function can be represented in slope intercept form as follows:

y = mx + b

where

m = slopeb = y-intercept

Therefore, to find the missing values in the table we have to use the equation of the linear function.

y = mx + b

let's find m.

using (4, -16) and (8, -36)

m = slope = -36 + 16 / 8 - 4

m = -20 / 4

m = -5

Hence, let's find y-intercept using (4, -16).

y = -5x + b

-16 = -5(4) + b

-16 + 20 = b

b = 4

Therefore, the equation is y = -5x + 4.

Let's find y₂, y₃ and y₄

y₂ = -5(5) + 4 = -25 + 4 = - 21

y₃ =  -5(6) + 4 = -30 + 4 = -26

y₄ =  -5(7) + 4 = -35 + 4 = - 31

learn more on linear function here: https://brainly.com/question/29491261

#SPJ1

1.
The band and the orchestra are each having a party to celebrate the
results of their latest competition. The band would like to order cupcakes
from Walmart at a cost of $3.35 per cupcake and a $13.00 order fee
The orchestra would like to order cupcakes from Publix at $2.90 each
and a $22.50 order fee. This system {2.90c +22.50 represents the
total cost, T, and the number of cupcakes. Determine how many
cupcakes each club can order if they both have the same amount of
money to spend. Is the solution viable or not viable?

Answers

By solving a linear equation, it can be calculated that

Each club can order 21.1 cupcakes if they both have the same amount of

money to spend

What is a linear equation?

Equation shows the comparison between two algebraic expressions by connecting the two algebraic expressions by an equal to sign

A one degree equation is called linear equation

Let the number of cupcakes ordered by each club be c

The band would like to order cupcakes from Walmart at a cost of $3.35 per cupcake and a $13.00 order fee

Total amount spent = $(13 + 3.35c)

The orchestra would like to order cupcakes from Publix at $2.90 each

and a $22.50 order fee

Total amount spent = $(22.5 + 2.90c)

By the problem,

The linear equation is

13 + 3.35c = 22.5 + 2.90c

3.35c - 2.90c = 22.5 - 13

0.45c = 9.5

c = 21.1

This solution is not viable as the number of cupcakes must be an integer not a decimal.

To learn more about linear equation , refer to the link-

https://brainly.com/question/2030026

#SPJ1

∠Cis a right angle and AB=14 miles, determine the length of BC and AC.

Answers

The length of BC is approximately 5.72 miles and the length of AC is approximately 12.67 miles.

Let's represent the length of the side opposite angle A "BC" and the length of the side adjacent to angle A "AC". We can then use the sine function to find BC:

sin(A) = BC / AC

Substituting the given values, we get:

sin(24°) = BC / 14

We can use a calculator or look up the value of sin(24°) to get:

sin(24°) ≈ 0.406

Solving for BC, we get:

BC = 14 * sin(24°) ≈ 5.72

To find AC, we can use the cosine function:

cos(A) = AC / AC

Substituting the given values, we get:

cos(24°) = AC / 14

We can use a calculator or look up the value of cos(24°) to get:

cos(24°) ≈ 0.913

Solving for AC, we get:

AC = 14 x cos(24°) ≈ 12.67

Thus, BC is about 5.72 miles long while AC is about 12.67 miles in length.

Learn more about the triangles here:

https://brainly.com/question/17997149

#SPJ1

a line passes through point (1, 6) and has a slope of 2. what is the y-intercept?

Answers

A line passes through point (1, 6) and has a slope is y=8x-14 We want to write the equation of the line that passes through (1, -6) and has a slope of 8 in slope-intercept form

What is equation?

The definition of an equation in algebra is a mathematical statement that demonstrates the equality of two mathematical expressions. For instance, the equation 3x + 5 = 14 consists of the two equations 3x + 5 and 14, which are separated by the 'equal' sign.

Slope-intercept form is given as y=mx+b, where m is the slope and b is the y intercept

As we are already given the slope (8), we can immediately plug that into the equation

So far, the equation of the line is:

y=8x+b

Now we need to find b

The line passes through the point (1, -6), which means that the point is a solution to the equation; when plugged into the equation, it'll create a true statement. This can help us find unknown variables, like b

Substitute 1 as x and -6 as y in the equation to solve for b

-6=8(1)+b

Multiply

-6=8+b

Subtract 8 from both sides

-14=b

Substitute -14 as b into the equation

y=8x-14

To learn more about equation visit

https://brainly.com/question/29657988

#SPJ1

Help!!!! Please Help Me!!! I need the answer right now!!! Thanks!!!

The subject is Science and Not Mathematics.
I couldn't find the Science option.

Answers

The numbers of the atoms of each element are;

Carbon dioxide;

6 atoms of carbon, 12 atoms of oxygen

Water ;

12 atoms of hydrogen, 6 atoms of oxygen

Glucose;

6 atoms of carbon, 12 atoms of hydrogen, 6 atoms of oxygen

Oxygen molecule

12 atoms of oxygen

What is a reaction?

In a chemical reaction, there is the combination of the atoms of the reactants and a consequent recombination as the products are formed. In accordance with the law of the conservation of mass, we know that the number of atoms before reaction must be equal to the number of atoms after reaction.

Now we have to look at the atoms and count up the number of atoms of each element in each of the compounds and this is how we are able to obtain the number of each atom as shown above.

Learn more about reaction:https://brainly.com/question/28984750

#SPJ1

The equation 4(2x 1) = 6x − 12 models the change in water level in a lake in one month. solve for x, which is change in height in inches. x equals negative thirteen halves x = −8 x = 8 x equals eleven halves

Answers

From the given options, option 2. x= -8 is the correct option. Here, the x is change in height in inches so the, change in height is -8 inches.

A linear equation is simply an equation with the highest degree always being 1. The standard form of a linear equation in one variable is Ax + B = 0. x is a variable in this equation, and A and B are constants. A two-variable linear equation has the standard form Ax + By = C. The variables x and y are variables, their coefficients A and B are variables, and C is a constant.

Given, linear equation is

[tex]4(2x+1)=6x-12\\\\8x+4=6x-12\\\\8x-6x=-4-12\\\\2x=-16\\\\x=-8[/tex]

Hence, the x equals to -8.

To learn more about linear equation refer here

https://brainly.com/question/29739212

#SPJ4

Other Questions
sports teams travel together. they compete against other teams. this increases their a.diversity. b.norm generalization. c.role structure. d.cohesiveness. e.informal leadership. a chemical called endogenous pyrogens may be released from lymphocytes that are responding to a pathogen. what does this compound cause? multiple choice fever activation of nk cells antibody production inflammation Lim and Zebrack (2004) reported that for family caregivers of dementia, having social support is significantly associated with improved : Help please!!! a) what are the s and n axes on an s-n diagram, and what does the region to the left and below the plotted line indicate? A train is travelling along its railroad tracks. It slows down, goes around a turn, speeds back up, coasts at a constant velocity for awhile, then slows down again.How many times did the train accelerate?Responses1234A train is travelling along its railroad tracks. It slows down, goes around a turn, speeds back up, coasts at a constant velocity for awhile, then slows down again.How many times did the train accelerate?Responses1234 which of the following is the best example of shaping? responses a child receives five dollars each time he cleans his room. a child receives five dollars each time he cleans his room. an employee receives a termination notice after coming to work late every day over a period of three months. an employee receives a termination notice after coming to work late every day over a period of three months. a child gets candy from a dispenser one time but gets nothing from the dispenser the next two times. a child gets candy from a dispenser one time but gets nothing from the dispenser the next two times. a teacher rewards a student for sitting quietly for ten minutes on monday, fifteen minutes on tuesday, twenty minutes on wednesday, and thirty minutes on thursday. a teacher rewards a student for sitting quietly for ten minutes on monday, fifteen minutes on tuesday, twenty minutes on wednesday, and thirty minutes on thursday. a rat receives a mild shock each time it tries to open the door of its cage. Now drag the Low risk and High risk project points so their expected rates of return are 7% and 11%, respectively. If you could choose only one project to go forward, which would you choose? a. Project Low because its expected rate of return is close to its WACC. b. Project Average because its expected rate of return equals its WACC and the others are both below their respective WACC's. c. Project High because its expected rate of return is higher than for any of the other projects. d. Any are good choices because the WACC balances the risk. peripheral tolerance group of answer choices is caused by costimulation without tcr-mhc peptide interaction. is caused by tcr-mhc peptide interaction without costimulation. leads to anergy. both leads to anergy and is caused by costimulation without tcr-mhc peptide interaction. both leads to anergy and is caused by tcr-mhc peptide interaction without costimulation. 14. Which of these statements correctly describes the function y = 4x + 9?A. It is not a linear function, and its graph is not a straight line.B. It is a linear function, but its graph is not a straight line.C. It is not a linear function, but its graph is a straight line.D. It is a linear function, and its graph is a straight line. Complete the below table to calculate the price of a $1 million bond issue under each of the following independent assumptions (FV of $1, PV of $1, FVA of $1, PVA of $1, FVAD of $1 and PVAD of $1) (Use appropriate factor(s) from the tables provided. Enter your answers in whole dollars.):1.Maturity 10 years, interest paid annually, stated rate 10%, effective (market) rate 12%Amount Present ValueInterestPrinciplePrice of bond2.Maturity 10 years, interest paid semiannually, stated rate 10%, effective (market) rate 12%Amount Present ValueInterestPrinciplePrice of bond3.Maturity 10 years, interest paid semiannually, stated rate 12%, effective (market) rate 10%Amount Present ValueInterestPrinciplePrice of bond4.Maturity 20 years, interest paid semiannually, stated rate 12%, effective (market) rate 10%Amount Present ValueInterestPrinciplePrice of bond5.Maturity 20 years, interest paid semiannually, stated rate 12%, effective (market) rate 12%A mount Present ValueInterestPrinciplePrice of bond which sentence is punctuated correctly? question 4 options: a) chris researched, visited, and booked the location, zach hired and supervised the caterers, and stephanie ordered and sent the invitations. b) chris researched, visited, and booked the location; zach hired and supervised the caterers; and stephanie ordered and sent the invitations. 2._____________This mountain range forms a natural boundary between France and Spain. The left and right cervical plexuses are located deep on each side of the neck, immediately lateral to cervical vertebrae ______. presented below is selected information for sheridan company. answer the questions asked about each of the factual situations. (do not leave any answer field blank. enter 0 for amounts.) 1. sheridan purchased a patent from vania co. for $1,160,000 on january 1, 2018. the patent is being amortized over its remaining legal life of 10 years, expiring on january 1, 2028. during 2020, sheridan determined that the economic benefits of the patent would not last longer than 6 years from the date of acquisition. what amount should be reported in the balance sheet for the patent, net of accumulated amortization, at december 31, 2020? Using the same country you chose for the mapping task as your subject, research important demographic factors using the World Factbook. Create a country profile, making sure to include the following information:gross domestic product per capitalife expectancyliteracy rateinfant mortality ratemajor agricultural productsmajor industriesmain exports and importsUsing this information, write a short profile of the country you chose. AUSTRAILA the east german government announced that its citizens could travel to west germany and west berlin, and the berlin wall opened. t/f two different two-digit whole numbers are selected at random. what is the probability that their product is less than 200. express your answer as a common fraction. (hints: (l) there are 90 different two-digit numbers, (2) the pair {10, 11} produces the smallest product and the pair {11, 18} produces the largest product less than 200). suppose you wish to run two different operating systems on one server. you can accomplish this by using . on the first line, ask the user how many days they've been driving for and declare the user input. it's an integer, so cast the string. After conquering much of Southeast Asia and the western Pacific, Japan proclaimed the organization of theA. Asian Popular Front.B. Great East Asia Co-Prosperity Sphere.C. Monroe Doctrine for Asia.D. Yankee Go Home Alliance.E. Asia for the Asians.B. Great East Asia Co-Prosperity Sphere